Free NCLEX practice review exam questions and answers with reasoning/explanation. Actual NCLEX questions and problems. Free NCLEX tutorial. Free NCLEX practice review exam with video. Different NCLEX practice questions. Automatic. Easy to Use.

Answer questions: START HERE | Psychotic Disorders | Substance Abuse, Eating Disorders, Impulse Control Disorders | OTHERS

Review for NCLEX-RN Examination 901-1000

--> QUESTION NUMBER _ 901 _ about (MC)


QUESTION: "For a neurologically injured client, the nurse would best assess motor strength by:"

CHOICES

( O ) a.) comparing equality of hand grasps.

( X ) b.) observing spontaneous movements.

( X ) c.) observing the client feed himself.

( X ) d.) asking him to signal if he feels pressure applied to his feet.


RATIONALE: Comparing equality of hand grasps is a technique used to assess motor strength. The ability to move spontaneously demonstrates motor ability but not strength. Noting that the client can feed himself verifies coordination and motor ability but does not help determine muscle strength. Having the client signal when pressure is applied to his feet tests sensory function. NURSING PROCESS STEP: Assessment CLIENT NEEDS CATEGORY: Physiological integrity CLIENT NEEDS SUBCATEGORY: Physiological adaptation
******************************

--> QUESTION NUMBER _ 902 _ about (MC)


QUESTION: "A client is suffering from short-term memory loss. Which of the following nursing actions would be appropriate to help him cope with his memory loss?"

CHOICES

( X ) a.) Instruct family members to ignore his behavior.

( O ) b.) Place a single-date calendar where he can view it.

( X ) c.) Tell him every morning what activities he will be expected to perform that day.

( X ) d.) Explain that he will have to try harder to remember things.


RATIONALE: It is not unusual for a client to be disoriented and suffer short-term memory loss after a head injury. Clocks, single-date calendars, and other items to help orient the client should be provided. Frequent reassurance and orientation by the nurse and family members will help the client understand the reason for his hospitalization and recognize that he is in a safe environment. Ignoring the client's behavior would not provide the client with the reassurance and assistance that he needs. Explanations of activities should be simple and given immediately before the procedure. Telling the client to try harder will increase his frustration and cause anxiety. NURSING PROCESS STEP: Implementation CLIENT NEEDS CATEGORY: Physiological integrity CLIENT NEEDS SUBCATEGORY: Physiological adaptation
******************************

--> QUESTION NUMBER _ 903 _ about (MC)


QUESTION: "The nurse enters the client's room as the client, who is sitting in a chair, begins to have a seizure. Which of the following actions should the nurse take first?"

CHOICES

( X ) a.) Lift the client onto his bed.

( O ) b.) Ease the client to the floor.

( X ) c.) Restrain the client's body movements.

( X ) d.) Insert an airway into the client's mouth.


RATIONALE: If a client has a seizure while sitting in a chair, it would be best to ease him to the floor and place a pillow under his head. The nurse, as well as the client, may be injured if the nurse tries to lift him to a bed. No effort should be made to restrain him. The strong muscle contractions may cause the restrained client to injure himself. Placing an airway in the client's mouth during a seizure is not necessary or recommended. NURSING PROCESS STEP: Implementation CLIENT NEEDS CATEGORY: Physiological integrity CLIENT NEEDS SUBCATEGORY: Physiological adaptation
******************************

--> QUESTION NUMBER _ 904 _ about (MC)


QUESTION: "Which of the following interventions will assist the client in taking phenytoin as prescribed?"

CHOICES

( X ) a.) Calling him daily for the first week after hospital discharge.

( X ) b.) Having a family member monitor him to ensure compliance.

( O ) c.) Providing him with written and verbal instructions about the medicine.

( X ) d.) Emphasizing that embarrassing seizures may occur again if he does not take the medicine.


RATIONALE: Providing the client with written and verbal instructions will increase understanding of the medication regimen. Calling the client daily should not be necessary to ensure compliance. The client should be responsible for taking his own medication and not rely on a family member. Reinforcing that seizures may be embarrassing is not an appropriate approach to improve medication compliance. NURSING PROCESS STEP: Implementation CLIENT NEEDS CATEGORY: Physiological integrity CLIENT NEEDS SUBCATEGORY: Pharmacological and parenteral therapies
******************************

--> QUESTION NUMBER _ 905 _ about (MC)


QUESTION: "A client is admitted with symptoms of a cerebrovascular accident. Nursing assessment data include: inability to move the right arm and leg, absence of muscle tone in the right arm and leg, and lack of knowledge about how to turn in bed. Based on these data, which of the following would be the most appropriate nursing diagnosis for this client?"

CHOICES

( X ) a.) Activity Intolerance.

( X ) b.) Disturbed Sleep Pattern.

( O ) c.) Impaired Physical Mobility.

( X ) d.) Unilateral Neglect.


RATIONALE: Based on the data provided, the most appropriate nursing diagnosis is Impaired Physical Mobility. It is plausible that the client will have difficulty tolerating activity; however, the data provided are specific to issues of mobility. There are no data to indicate that this client is suffering from a disturbance in sleep patterns. While the client may develop Unilateral Neglect, based on the current data there is no indication that the client is neglecting the right side of his body at this time. NURSING PROCESS STEP: Analysis, nursing diagnosis CLIENT NEEDS CATEGORY: Physiological integrity CLIENT NEEDS SUBCATEGORY: Basic care and comfort
******************************

--> QUESTION NUMBER _ 906 _ about (MC)


QUESTION: "A client has had a cerebrovascular accident (CVA). Because the CVA affected the left side of the client's brain, the nurse should anticipate that the client would most likely experience:"

CHOICES

( O ) a.) Expressive aphasia.

( X ) b.) Dyslexia.

( X ) c.) Apraxia.

( X ) d.) Agnosia.


RATIONALE: Broca's area, which controls expressive speech, is located on the left side of the brain. Therefore, a client with a cerebrovascular accident in this area is likely to exhibit expressive or motor aphasia. Dyslexia, the inability of a person with normal vision to interpret written language, is thought to be due to a central nervous system defect in the ability to organize graphic symbols. Apraxia is the inability to perform purposeful movements in the absence or loss of motor power, sensation, or coordination. Agnosia is the loss of comprehension of auditory, visual, or other sensations despite an intact sensory sphere. NURSING PROCESS STEP: Planning CLIENT NEEDS CATEGORY: Physiological integrity CLIENT NEEDS SUBCATEGORY: Physiological adaptation
******************************

--> QUESTION NUMBER _ 907 _ about (MC)


QUESTION: "When helping the client who has had a cerebrovascular accident (CVA) learn self-care skills, the nurse should use which of the following interventions to help him learn to dress himself?"

CHOICES

( X ) a.) Encourage the client to wear clothing designed especially for people who have had a CVA.

( X ) b.) Dress the client, explaining each step of the process as it is completed.

( O ) c.) Teach the client to put on clothing on the affected side first.

( X ) d.) Encourage the client to ask his wife for help when dressing.


RATIONALE: When dressing, the client will find it easier to dress if clothing is put on the affected side first. Stoke clients should wear normal clothing, if possible. Dressing the client does not promote self-care skills. Other people may help the client dress, but the emphasis should be on self-care. NURSING PROCESS STEP: Implementation CLIENT NEEDS CATEGORY: Physiological integrity CLIENT NEEDS SUBCATEGORY: Basic care and comfort
******************************

--> QUESTION NUMBER _ 908 _ about (MC)


QUESTION: "The nurse is preparing the client with a cerebrovascular accident for discharge to home. Which of the following factors would most likely influence the client's continuing progress in rehabilitation at home?"

CHOICES

( O ) a.) Family's ability to provide support to the client.

( X ) b.) Client's ability to ambulate.

( X ) c.) Availability of a home health aide to care for the client.

( X ) d.) Frequency of follow-up visits with the physician.


RATIONALE: The strong support of family members is frequently identified as an important factor that influences a stroke client's continuing progress in rehabilitation after discharge. Discharge planning should prepare the client and family for the many changes necessary when the client returns home. Continuing progress in rehabilitation is not dependent upon the client's ability to ambulate. A client's continuing progress in rehabilitation is more dependent on the client and family support than the home health aide. Follow-up visits with the physician are important but are not the greatest influence on the client's progress with rehabilitation. NURSING PROCESS STEP: Planning CLIENT NEEDS CATEGORY: Psychosocial integrity CLIENT NEEDS SUBCATEGORY: Coping and adaptation
******************************

--> QUESTION NUMBER _ 909 _ about (MC)


QUESTION: "When assessing the client with Parkinson's disease, the nurse would anticipate which of the following signs and symptoms?"

CHOICES

( X ) a.) Dry mouth.

( X ) b.) Aphasia.

( X ) c.) An exaggerated sense of euphoria.

( O ) d.) A stiff, mask-like facial expression.


RATIONALE: Typical signs of Parkinson's disease include drooling; a low-pitched, monotonous voice; and a stiff, mask-like facial expression. Dry mouth is not associated with Parkinson's disease. Aphasia is not a symptom of Parkinson's disease. An exaggerated sense of euphoria would not be typical; more likely, the client would exhibit depression, probably related to the progressive nature of the disease and the client's difficulties dealing with it. NURSING PROCESS STEP: Assessment CLIENT NEEDS CATEGORY: Physiological integrity CLIENT NEEDS SUBCATEGORY: Physiological adaptation
******************************

--> QUESTION NUMBER _ 910 _ about (MC)


QUESTION: "When developing a plan of home care for the client with multiple sclerosis, the nurse should teach the client about which of the following complications most likely to occur?"

CHOICES

( X ) a.) Ascites.

( O ) b.) Contractures.

( X ) c.) Fluid volume overload.

( X ) d.) Myocardial infarction.


RATIONALE: Typical complications of multiple sclerosis include contractures, decubitus ulcers, and respiratory infections. Nursing care should be directed toward the goal of preventing these complications. Ascites is not associated with multiple sclerosis. Fluid volume overload is not associated with multiple sclerosis. Myocardial infarction is not associated with multiple sclerosis. NURSING PROCESS STEP: Planning CLIENT NEEDS CATEGORY: Physiological integrity CLIENT NEEDS SUBCATEGORY: Basic care and comfort
******************************

--> QUESTION NUMBER _ 911 _ about (MC)


QUESTION: "The client with multiple sclerosis sometimes exhibits signs or symptoms of emotional distress. The nurse should be aware that clients with multiple sclerosis are most likely to exhibit:"

CHOICES

( O ) a.) mood disorders.

( X ) b.) thought disorders.

( X ) c.) psychosomatic illnesses.

( X ) d.) drug dependency problems.


RATIONALE: Clients with multiple sclerosis often experience psychological disturbances that are best described as mood disorders. Emotional instability is typical. Thought disorders are not typical of clients with multiple sclerosis unless they present independently. Psychosomatic illnesses are not typical of clients with multiple sclerosis unless they present independently. Drug dependency is not typical of clients with multiple sclerosis unless it presents independently. NURSING PROCESS STEP: Planning CLIENT NEEDS CATEGORY: Physiological integrity CLIENT NEEDS SUBCATEGORY: Physiological adaptation
******************************

--> QUESTION NUMBER _ 912 _ about (MC)


QUESTION: "Which of the following assessment findings would lead the nurse to suspect that the victim of an automobile accident is comatose as a result of a toxic drug overdose?"

CHOICES

( X ) a.) Hypertension.

( X ) b.) Fever.

( O ) c.) Constricted pupils.

( X ) d.) Warm, dry skin.


RATIONALE: Equal, normally reactive pupils indicate adequate neurologic functioning. An overdose of narcotics or depressants can cause constricted pupils. Blood pressure is regulated by various factors. A finding of hypertension would not pinpoint a toxic disorder. Fever is related either to infection or dehydration. Clients with a drug overdose have cold, clammy skin. NURSING PROCESS STEP: Assessment CLIENT NEEDS CATEGORY: Physiological integrity CLIENT NEEDS SUBCATEGORY: Pharmacological and parenteral therapies
******************************

--> QUESTION NUMBER _ 913 _ about (MC)


QUESTION: "The comatose victim of the car accident is to have a gastric lavage. Which of the following positions would be most appropriate for the client during this procedure?"

CHOICES

( O ) a.) Lateral.

( X ) b.) Supine.

( X ) c.) Trendelenburg's.

( X ) d.) Lithotomy.


RATIONALE: An unconscious client is best positioned in a lateral or semiprone position because these positions allow the jaw and tongue to fall forward, facilitate drainage of secretions, and prevent aspiration. Positioning the client supine carries a major risk of airway obstruction from the tongue, vomitus, or nasopharyngeal secretions. Trendelenburg's position, with the head lower than the heart, decreases effective lung volume and increases the risk of cerebral edema. The lithotomy position has no purpose in this situation. NURSING PROCESS STEP: Implementation CLIENT NEEDS CATEGORY: Physiological integrity CLIENT NEEDS SUBCATEGORY: Reduction of risk potential
******************************

--> QUESTION NUMBER _ 914 _ about (MC)


QUESTION: "A client is at risk for developing a pressure ulcer. The first warning of an impending pressure ulcer is when pressure applied to skin turns it:"

CHOICES

( X ) a.) Bluish.

( O ) b.) Reddish.

( X ) c.) Whitish.

( X ) d.) Yellowish.


RATIONALE: Techniques for staging of pressure ulcers can be found in the Guidelines for Pressure Ulcers published by the Agency for Healthcare Research and Quality (AHRQ), a division of the Department of Health and Human Services. When pressure is applied to the skin, the area first becomes blanched, or whitish. When pressure is relieved, the circulation tends to carry excess blood to the area to make up for the temporary decrease in blood supply. This effect, called reactive hyperemia, causes the skin to redden. Such a reddened area is a precursor of a pressure ulcer. Bluish skin indicates a lack of oxygen in the tissues, not the development of a pressure ulcer. Whitish skin may be indicative of arterial insufficiency to an area. A yellowish cast when blanched suggests jaundice, which indicates a liver or biliary tract disorder. NURSING PROCESS STEP: Assessment CLIENT NEEDS CATEGORY: Physiological integrity CLIENT NEEDS SUBCATEGORY: Physiological adaptation
******************************

--> QUESTION NUMBER _ 915 _ about (MC)


QUESTION: "To prevent external rotation of the client's hips while he is lying on his back, it would be best for the nurse to place:"

CHOICES

( X ) a.) Firm pillows under the length of his legs.

( X ) b.) Sandbags alongside his legs from knees to ankles.

( O ) c.) Trochanter rolls alongside his legs from ilium to midthigh.

( X ) d.) A footboard that supports his feet in the normal anatomic position.


RATIONALE: Trochanter rolls placed alongside the client's legs from the ilium to midthigh are recommended to prevent external rotation of the hips. Pillows can be used only as a temporary measure because they cannot hold the legs and hips in proper alignment over a prolonged period. Placing sandbags from the knees to the ankle will not effectively support the hips in proper alignment. A footboard does not help to keep the legs and hips in proper alignment. NURSING PROCESS STEP: Implementation CLIENT NEEDS CATEGORY: Physiological integrity CLIENT NEEDS SUBCATEGORY: Physiological adaptation
******************************

--> QUESTION NUMBER _ 916 _ about (MC)


QUESTION: "After 5 days of hospitalization, the client who is receiving morphine sulfate for pain control asks for pain medication with increasing frequency and exhibits increased anxiety and restlessness. His physical condition is stable. What is the probable cause of his behavior?"

CHOICES

( X ) a.) His morphine dosage is too high.

( X ) b.) He is becoming addicted to the narcotic.

( X ) c.) His coping mechanisms are exhausted.

( O ) d.) He has developed tolerance to his narcotic dosage.


RATIONALE: Tolerance to a regular narcotic dose can develop with frequent use. The client experiences increased discomfort, asks for medication more frequently, and exhibits anxious and restless behavior, which are often misinterpreted as indicative of developing dependence or addiction. The client's symptoms do not suggest that the dosage is too high. Addiction is a psychological condition in which a client takes drugs for non-therapeutic reasons. This client is receiving morphine for pain control. There are no data presented about the client's coping mechanisms. NURSING PROCESS STEP: Assessment CLIENT NEEDS CATEGORY: Physiological integrity CLIENT NEEDS SUBCATEGORY: Pharmacological and parenteral therapies
******************************

--> QUESTION NUMBER _ 917 _ about (MC)


QUESTION: "Research studies have demonstrated that client-controlled analgesia is more effective than intermittent narcotic administration. Which response by the nurse to a client asking about client-controlled analgesia reflects knowledge of these research findings? "Client-controlled analgesia is more effective because:"

CHOICES

( X ) a.) A different narcotic is used."

( X ) b.) Two narcotics are administered simultaneously."

( O ) c.) The client controls the amount of pain medication administered."

( X ) d.) The nurse interrupts the client less frequently, and the client can get more sleep."


RATIONALE: Studies have supported that one reason client-controlled analgesia is effective is because the client has control over the narcotic administration. Morphine is the most commonly used narcotic in client-controlled analgesia. Only one narcotic is administered at a time. Nursing assessments and actions remain basically the same for the client using client-controlled analgesia. NURSING PROCESS STEP: Implementation CLIENT NEEDS CATEGORY: Physiological integrity CLIENT NEEDS SUBCATEGORY: Pharmacological and parenteral therapies
******************************

--> QUESTION NUMBER _ 918 _ about (MC)


QUESTION: "Which of the following approaches to chronic pain management is most effective?"

CHOICES

( X ) a.) Physiologic approach.

( X ) b.) Psychological approach.

( X ) c.) Pharmacologic approach.

( O ) d.) Multidisciplinary approach.


RATIONALE: A multidisciplinary approach to pain relief is needed for greatest effectiveness. In addition to the client, the nurse, and the physician, others who may be needed on the team include a social worker, an occupational therapist, a dietitian, and a psychologist or a psychiatrist. Pain relief interventions based on physiologic principles can be used simultaneously to obtain greater pain relief. Pain relief interventions based on psychological principles can be used simultaneously to obtain greater pain relief. Medication administration is only one option for reducing pain. NURSING PROCESS STEP: Evaluation CLIENT NEEDS CATEGORY: Safe, effective care environment CLIENT NEEDS SUBCATEGORY: Management of care
******************************

--> QUESTION NUMBER _ 919 _ about (MC)


QUESTION: "The client with rheumatoid arthritis asks the nurse to explain why her joints are becoming increasingly painful. The nurse's response should be based on knowledge that rheumatoid arthritis:"

CHOICES

( X ) a.) Results from degenerative joint damage.

( X ) b.) Affects only the weight-bearing joints of the body.

( O ) c.) Begins with inflammation of joint synovial tissue.

( X ) d.) Is usually caused by aging.


RATIONALE: Synovial joints are characteristically affected by rheumatoid arthritis. Rheumatoid arthritis is an inflammatory disorder that most commonly affects middle-aged women. Osteoarthritis is a degenerative joint disease that affects weight-bearing joints. Degenerative joint disease occurs in osteoarthritis. Osteoarthritis is associated with aging. NURSING PROCESS STEP: Implementation CLIENT NEEDS CATEGORY: Physiological integrity CLIENT NEEDS SUBCATEGORY: Physiological adaptation
******************************

--> QUESTION NUMBER _ 920 _ about (MC)


QUESTION: "The client with rheumatoid arthritis has been taking large doses of aspirin to relieve her joint pain. The nurse should assess the client for which important symptom of aspirin toxicity?"

CHOICES

( X ) a.) Dysuria.

( O ) b.) Tinnitus.

( X ) c.) Chest pain.

( X ) d.) Drowsiness.


RATIONALE: Tinnitus (ringing in the ears) is a common symptom of aspirin toxicity. Dysuria is not associated with aspirin toxicity. Chest pain is not associated with aspirin toxicity. Drowsiness is not associated with aspirin toxicity. NURSING PROCESS STEP: Assessment CLIENT NEEDS CATEGORY: Physiological integrity CLIENT NEEDS SUBCATEGORY: Pharmacological and parenteral therapies
******************************

--> QUESTION NUMBER _ 921 _ about (MC)


QUESTION: "Which statement by the client with rheumatoid arthritis would indicate that she needs additional teaching to safely receive the maximum benefit of her aspirin therapy?"

CHOICES

( X ) a.) "I always take aspirin with food to protect my stomach."

( X ) b.) "Once I learned to take my aspirin with meals, I was able to start using the inexpensive generic brand."

( X ) c.) "I always watch for bleeding gums or blood in my stool."

( O ) d.) "I try to take aspirin only on days when the pain seems particularly bad."


RATIONALE: Aspirin therapy in rheumatoid arthritis involves continuous ongoing administration to establish and maintain therapeutic blood levels. Aspirin should not be used on an as-needed basis. Aspirin should always be buffered with food. Generic aspirin is acceptable. Clients should be instructed to observe for symptoms of bleeding. NURSING PROCESS STEP: Evaluation CLIENT NEEDS CATEGORY: Physiological integrity CLIENT NEEDS SUBCATEGORY: Pharmacological and parenteral therapies
******************************

--> QUESTION NUMBER _ 922 _ about (MC)


QUESTION: "Which of the following activities would the nurse likely choose to implement in response to a nursing diagnosis of Activity Intolerance related to lack of energy conservation?"

CHOICES

( X ) a.) Encourage the client to perform all tasks early in the day.

( O ) b.) Encourage the client to alternate periods of rest and activity throughout the day.

( X ) c.) Administer narcotics to promote pain relief and rest.

( X ) d.) Instruct the client to not perform daily hygienic care until activity tolerance improves.


RATIONALE: The client with rheumatoid arthritis should be encouraged to alternate periods of activity and rest throughout the day. Encouraging the client to perform all activities of daily living at once will worsen fatigue and stress her ability to recover. Narcotics are not typically administered to control arthritic pain. Encouraging the client to cease all participation in daily activities will decrease activity tolerance and make fatigue more pronounced. NURSING PROCESS STEP: Implementation CLIENT NEEDS CATEGORY: Physiological integrity CLIENT NEEDS SUBCATEGORY: Basic care and comfort
******************************

--> QUESTION NUMBER _ 923 _ about (MC)


QUESTION: "In developing a plan of care for a client with rheumatoid arthritis, the nurse should consider that clients with rheumatoid arthritis should be positioned so as to:"

CHOICES

( X ) a.) Decrease edema around the joints.

( X ) b.) Promote maximum comfort.

( X ) c.) Prevent venous stasis.

( O ) d.) Prevent flexion deformities of the joints.


RATIONALE: Proper positioning to prevent flexion deformities of the joints is an ongoing need for clients with rheumatoid arthritis and should be included in the care plan. Joints may be inflamed and swollen, but are not edematous. Positioning to promote comfort is not unique to clients with rheumatoid arthritis. Avoiding venous stasis is important but not unique to clients with rheumatoid arthritis. NURSING PROCESS STEP: Planning CLIENT NEEDS CATEGORY: Physiological integrity CLIENT NEEDS SUBCATEGORY: Basic care and comfort
******************************

--> QUESTION NUMBER _ 924 _ about (MC)


QUESTION: "A client undergoes a total right knee replacement. The nurse would anticipate which of the following activity orders for this client on the first postoperative day?"

CHOICES

( X ) a.) Bed rest for 24 to 48 hours after surgery.

( X ) b.) Ambulate with walker twice a day.

( O ) c.) Up to chair with leg elevated.

( X ) d.) Dangle at bedside for 20 minutes.


RATIONALE: Usual postoperative activity orders for a client with a total knee replacement include transferring the client out of bed to a chair on the first postoperative day. The affected leg is protected with a knee immobilizer and elevated while the client is up in the chair. Bed rest is unnecessary and increases risks of thrombophlebitis. Activity will progress to ambulating with a walker and assistance after the client is first up in a chair. Activity progresses to partial weight bearing with the use of assistive devices. NURSING PROCESS STEP: Planning CLIENT NEEDS CATEGORY: Physiological integrity CLIENT NEEDS SUBCATEGORY: Physiological adaptation
******************************

--> QUESTION NUMBER _ 925 _ about (MC)


QUESTION: "After a right total knee replacement, the client's right leg is placed in a continuous passive motion (CPM) machine. Nursing responsibilities when caring for a client with this apparatus would include which of the following?"

CHOICES

( X ) a.) Adjusting the settings as needed to prevent client discomfort.

( X ) b.) Increasing the range-of-motion settings at least every 8 hours.

( O ) c.) Maintaining proper positioning of the leg on the CPM machine.

( X ) d.) Discontinuing the CPM therapy when the client's range-of-motion increases.


RATIONALE: The nurse must frequently evaluate the positioning of the client's leg, the range-of-motion setting, and the client's response to the therapy. Using a CPM machine will likely produce initial discomfort for the client. If the client cannot tolerate the discomfort, the physician should be notified for an order to adjust the settings. The settings for the machine are determined by the physician and cannot be changed without an order. Therapy will continue until the client regains 90 degrees flexion in the knee. NURSING PROCESS STEP: Implementation CLIENT NEEDS CATEGORY: Physiological integrity CLIENT NEEDS SUBCATEGORY: Reduction of risk potential
******************************

--> QUESTION NUMBER _ 926 _ about (MC)


QUESTION: "The nurse teaches the client to perform isometric exercises to strengthen her leg muscles after surgery. Isometric exercises are particularly effective for clients with rheumatoid arthritis because they:"

CHOICES

( X ) a.) Cost little in terms of time and money.

( O ) b.) Strengthen the muscles while keeping the joints stationary.

( X ) c.) Involve clients in their own care and thus improve morale.

( X ) d.) Prevent joint stiffness.


RATIONALE: An exercise program is recommended to strengthen muscles after arthroplasty. Isometric (or muscle-setting) exercises strengthen muscles but keep the joint stationary during the healing process. Isometric exercise costs little in terms of time and money but these are not necessarily primary reasons for using it. Isometric exercises may help improve a client's morale by promoting self-care but this is not the reason for doing them. Isometric exercise will not help prevent joint stiffness; the joint is kept stationary. NURSING PROCESS STEP: Implementation CLIENT NEEDS CATEGORY: Physiological integrity CLIENT NEEDS SUBCATEGORY: Physiological adaptation
******************************

--> QUESTION NUMBER _ 927 _ about (MC)


QUESTION: "Which of the following expected outcomes would be appropriate for a client with rheumatoid arthritis? The client will:"

CHOICES

( X ) a.) Minimize the frequency with which anti-inflammatory drugs are used to control joint discomfort.

( O ) b.) Demonstrate use of adaptive equipment in the home environment as appropriate.

( X ) c.) Learn to limit activity so as to avoid joint pain.

( X ) d.) Verbalize that recovery from rheumatoid arthritis will require several years of treatment.


RATIONALE: Depending on the degree of joint involvement, clients with rheumatoid arthritis may need to learn to function with adaptive equipment. Such equipment can help the client maintain independence. The consistent use of anti-inflammatory drugs is considered important to minimize joint inflammation and damage. Periods of activity should be alternated with rest periods, but limiting activity to avoid joint pain is not a realistic or desirable outcome. The client needs to understand that rheumatoid arthritis cannot be cured. NURSING PROCESS STEP: Evaluation CLIENT NEEDS CATEGORY: Physiological integrity CLIENT NEEDS SUBCATEGORY: Basic care and comfort
******************************

--> QUESTION NUMBER _ 928 _ about (MC)


QUESTION: "A client has been diagnosed with degenerative joint disease (osteoarthritis) of the left hip. Which of the following factors would most likely increase the joint symptoms of osteoarthritis?"

CHOICES

( X ) a.) A long history of smoking.

( X ) b.) Excessive alcohol use.

( O ) c.) Obesity.

( X ) d.) Emotional stress.


RATIONALE: Osteoarthritis most commonly results from "wear and tear"-excessive and prolonged mechanical stress on the joints. Increased weight increases stress on weight-bearing joints. Therefore, an obese client with osteoarthritis should be encouraged to lose weight. Smoking does not cause osteoarthritis. Excessive alcohol use does not cause osteoarthritis. Emotional stress does not cause osteoarthritis. NURSING PROCESS STEP: Assessment CLIENT NEEDS CATEGORY: Health promotion and maintenance CLIENT NEEDS SUBCATEGORY: None
******************************

--> QUESTION NUMBER _ 929 _ about (MC)


QUESTION: "The nurse would anticipate conservative treatment of osteoarthritis to include:"

CHOICES

( X ) a.) Isotonic exercises to strengthen muscles.

( X ) b.) Opioid analgesics for pain control.

( X ) c.) Routine injections of intra-articular corticosteroids.

( O ) d.) Orthotic devices to support involved joints.


RATIONALE: Orthotic devices such as braces or splints may be used to provide support to affected joints. Isometric, not isotonic, exercises are used to strengthen muscles. Opioid analgesics are not used for osteoarthritic pain control. Acetaminophen and selected nonsteroidal anti-inflammatory drugs may be used to achieve pain relief. Intra-articular corticosteroid injections are not used routinely; rather they are cautiously used during periods of acute joint pain. NURSING PROCESS STEP: Planning CLIENT NEEDS CATEGORY: Physiological integrity CLIENT NEEDS SUBCATEGORY: Physiological adaptation
******************************

--> QUESTION NUMBER _ 930 _ about (MC)


QUESTION: "The nurse would plan to use an abduction pillow (or splint) after a total hip replacement to:"

CHOICES

( X ) a.) Prevent hip flexion.

( X ) b.) Decrease formation of sacral pressure ulcers.

( O ) c.) Prevent dislocation of the prosthesis.

( X ) d.) Increase peripheral circulation.


RATIONALE: After a total hip replacement, it is important to maintain the hip in a state of abduction to prevent dislocation of the prosthesis. Use of an abduction pillow or splint will not prevent hip flexion. Use of an abduction pillow or splint will not prevent the formation of sacral pressure ulcers. Use of an abduction pillow or splint will not increase peripheral circulation. NURSING PROCESS STEP: Planning CLIENT NEEDS CATEGORY: Physiological integrity CLIENT NEEDS SUBCATEGORY: Reduction of risk potential
******************************

--> QUESTION NUMBER _ 931 _ about (MC)


QUESTION: "The client tells the nurse that the pain in his operative hip has increased. Assessing the hip and leg, the nurse notes that the leg is internally rotated and shorter than the other leg and that the client has difficulty moving the leg. Based on this information, the nurse determines that the client:"

CHOICES

( X ) a.) Has experienced increased pain due to a muscle spasm.

( X ) b.) Requires repositioning to achieve better alignment of the leg.

( X ) c.) Would benefit from additional muscle strengthening exercises.

( O ) d.) Has experienced a dislocation of the hip prosthesis.


RATIONALE: Classic signs of dislocation of the hip prosthesis include increasing pain, abnormal rotation, shortened leg, difficulty or inability moving the leg, and misalignment of the leg. The nurse should notify the surgeon so that the prosthesis can be repositioned. Muscle spasm will cause pain but would not be responsible for the internal rotation and shortening of the extremity. Repositioning the client will not help this situation. The surgeon will have to relocate the hip joint. Muscle strengthening exercises may help prevent subsequent dislocations, however, hip precautions must be maintained until complete healing has occurred. NURSING PROCESS STEP: Analysis CLIENT NEEDS CATEGORY: Physiological integrity CLIENT NEEDS SUBCATEGORY: Physiological adaptation
******************************

--> QUESTION NUMBER _ 932 _ about (MC)


QUESTION: "The nurse has instructed the client about the correct positioning of his leg and hip following hip replacement surgery. Which of the following statements indicate that the client has understood these instructions?"

CHOICES

( X ) a.) "I may cross my legs as long as I keep my knees extended."

( O ) b.) "I should avoid bending over to tie my shoes."

( X ) c.) "I can sit in any chair that I find comfortable."

( X ) d.) "I should avoid any unnecessary walking for about 3 months after my surgery."


RATIONALE: Acute flexion and adduction of the hip should be avoided after hip replacement surgery. The client may not cross (adduct) his legs as this puts him at risk for dislocating the prosthesis. The client should not sit in low chairs that will require excessive hip flexion to get in or out of. Hip flexion also increases the risk of dislocation. Frequent walks are encouraged to increase muscle strength and provide hip exercises. NURSING PROCESS STEP: Evaluation CLIENT NEEDS CATEGORY: Physiological integrity CLIENT NEEDS SUBCATEGORY: Basic care and comfort
******************************

--> QUESTION NUMBER _ 933 _ about (MC)


QUESTION: "A client is admitted to the hospital with a diagnosis of a right hip fracture. She complains of right hip pain and cannot move her right leg. Which of the following assessments made by the nurse indicates that the client has a typical sign of a hip fracture? The client's right leg is:"

CHOICES

( X ) a.) Rotated internally.

( X ) b.) Held in a flexed position.

( X ) c.) Adducted.

( O ) d.) Shorter than the leg on the unaffected side.


RATIONALE: After a hip fracture, the leg on the affected side is characteristically shorter than the unaffected leg. A fractured hip usually rotates externally. Holding the leg in a flexed position is seen in patients with a dislocated hip, not a fractured hip. Typically, the fractured hip is in an abducted position. NURSING PROCESS STEP: Assessment CLIENT NEEDS CATEGORY: Physiological integrity CLIENT NEEDS SUBCATEGORY: Physiological adaptation
******************************

--> QUESTION NUMBER _ 934 _ about (MC)


QUESTION: "Which of the following signs or symptoms would be of least importance when the nurse evaluates the client for postoperative peripheral nerve damage?"

CHOICES

( X ) a.) Pain.

( X ) b.) Altered sensation.

( O ) c.) Bleeding.

( X ) d.) Pulselessness.


RATIONALE: Neurovascular damage may be indicated by the presence of any of the "five Ps": pain, pallor, pulselessness, paresthesia, and paralysis. Bleeding does not indicate neurovascular damage. Neurovascular damage can occur after almost any orthopedic surgery. Pain can indicate nerve damage. A thorough evaluation of the pain should help distinguish if it is expected postoperative pain or is indicative of nerve damage. Altered sensation is indicative of nerve dysfunction and needs to be fully evaluated to determine cause. Decreased or absent pulses is indicative of vascular compromise and needs to be evaluated immediately. NURSING PROCESS STEP: Evaluation CLIENT NEEDS CATEGORY: Physiological integrity CLIENT NEEDS SUBCATEGORY: Physiological adaptation
******************************

--> QUESTION NUMBER _ 935 _ about (MC)


QUESTION: "Which of the following pieces of equipment should the nurse plan to use to help prevent external rotation of the client's right leg postoperatively?"

CHOICES

( O ) a.) Sandbags.

( X ) b.) A high footboard.

( X ) c.) A rubber air ring.

( X ) d.) A metal bed cradle.


RATIONALE: It is best to support the client's leg in its proper anatomic position and to prevent external rotation by supporting the leg with sandbags. A trochanter roll can also be used. Sandbags should be placed along the length of the thigh and lower leg. A footboard will not help prevent external rotation of the leg. A rubber air ring will not help prevent external rotation of the leg. A metal frame will not help prevent external rotation of the leg. NURSING PROCESS STEP: Planning CLIENT NEEDS CATEGORY: Physiological integrity CLIENT NEEDS SUBCATEGORY: Reduction of risk potential
******************************

--> QUESTION NUMBER _ 936 _ about (MC)


QUESTION: "Which of the following nursing measures would be most important to implement to decrease the risk of a surgical wound infection in a client with an internal fixation and hip pinning?"

CHOICES

( X ) a.) Inserting an indwelling urinary catheter to prevent possible soiling of the dressing.

( X ) b.) Accurately measuring drainage from the surgical drainage tube.

( O ) c.) Changing the surgical dressings using sterile technique.

( X ) d.) Monitoring the incision for signs of redness, swelling, and warmth.


RATIONALE: Wound infection can best be prevented by using strict sterile technique during dressing changes. Inserting a Foley catheter is an unnecessary action in this case and would predispose the client to a urinary tract infection. Accurately measuring drainage is an important nursing action but will not prevent a wound infection. Monitoring the incision for signs of infection is an important nursing action but will not prevent a wound infection. NURSING PROCESS STEP: Implementation CLIENT NEEDS CATEGORY: Physiological integrity CLIENT NEEDS SUBCATEGORY: Physiological adaptation
******************************

--> QUESTION NUMBER _ 937 _ about (MC)


QUESTION: "When the client who has had a hip replacement is lying on her side, the nurse should place pillows or an abductor splint between her legs to prevent:"

CHOICES

( X ) a.) Flexion of the knees.

( X ) b.) Abduction of the thighs.

( O ) c.) Adduction of the hip joint.

( X ) d.) Hyperextension of the knees.


RATIONALE: After hip replacement surgery, the client should be positioned on the non-operative side with pillows or an abductor splint between the legs to help prevent adduction of the operative leg. This positioning places the hip in proper alignment. Dislocation of the hip can occur if the leg on the affected side is allowed to adduct. Flexion of the knees is not contraindicated. Abduction of the legs is the correct position. Placing a pillow between the legs will not result in hyperextension of the knee. Hyperextension of the knee is to be avoided in any case because it can result in injury. NURSING PROCESS STEP: Implementation CLIENT NEEDS CATEGORY: Physiological integrity CLIENT NEEDS SUBCATEGORY: Reduction of risk potential
******************************

--> QUESTION NUMBER _ 938 _ about (MC)


QUESTION: "When assessing a client as a candidate for crutch walking, the nurse should take into account that for some elderly people, crutch walking is an impractical goal primarily because of decreased:"

CHOICES

( X ) a.) Visual acuity.

( X ) b.) Reaction time.

( O ) c.) Motor coordination.

( X ) d.) Level of comprehension.


RATIONALE: Some elderly people are not good candidates for crutch walking because they are not strong enough to use crutches or are not coordinated enough to walk safely with crutches. Visual acuity may influence the ability to learn crutch walking but is not as important as motor coordination. Reaction time may influence the ability to learn crutch walking but is not as important as motor coordination. Level of comprehension may influence the ability to learn crutch walking but is not as important as motor coordination. NURSING PROCESS STEP: Assessment CLIENT NEEDS CATEGORY: Physiological integrity CLIENT NEEDS SUBCATEGORY: Physiological adaptation
******************************

--> QUESTION NUMBER _ 939 _ about (MC)


QUESTION: "Which of the following activities should the nurse plan to teach the client to strengthen her hand muscles in preparation for using crutches?"

CHOICES

( X ) a.) Brushing her hair.

( O ) b.) Squeezing a rubber ball.

( X ) c.) Flexing and extending her wrists.

( X ) d.) Pushing her hands into the mattress while raising herself in bed.


RATIONALE: A client being prepared for crutch walking should be taught to support her weight with her hands when crutch walking. Supporting weight in the axillae is contraindicated owing to the risk of possible nerve damage and circulatory obstruction. The client should be taught to squeeze a ball vigorously to help strengthen her hands in preparation for weight bearing with the hands. Hair brushing is not likely to strengthen her hands. Wrist flexion and extension may help with wrist joint mobility but will not strengthen the hands. Using the hands to push up in bed may be helpful in maintaining the current level of strength. It will not help build strength unless done frequently and regularly as an exercise. NURSING PROCESS STEP: Planning CLIENT NEEDS CATEGORY: Physiological integrity CLIENT NEEDS SUBCATEGORY: Reduction of risk potential
******************************

--> QUESTION NUMBER _ 940 _ about (MC)


QUESTION: "A client is diagnosed with a herniated lumbar disk at the L-5 interspace. During the initial client interview, the nurse would most likely learn that the symptom that first caused the client to seek health care was:"

CHOICES

( X ) a.) Loss of voluntary muscle control.

( X ) b.) Loss of bladder control.

( O ) c.) Back pain that is relieved with resting.

( X ) d.) Back pain that radiates to the shoulders.


RATIONALE: A typical symptom of a herniated lumbar disk is low back pain that is usually relieved by rest and aggravated by activity that causes an increase in fluid pressure in the spine, such as sneezing, coughing, lifting, and bending. Muscle weakness and sensory losses may occur, and there is generally a change in tendon reflexes. Loss of voluntary muscle control is not typically an early symptom of lumbar disk problems. Loss of urinary control is not typically an early symptom of lumbar disk problems. Pain radiating to the shoulders, which often mimics some symptoms of a heart attack, is a typical symptom of cervical disk herniation. NURSING PROCESS STEP: Assessment CLIENT NEEDS CATEGORY: Health promotion and maintenance CLIENT NEEDS SUBCATEGORY: None
******************************

--> QUESTION NUMBER _ 941 _ about (MC)


QUESTION: "A client returns from a myelogram, for which an iodized oil (Pantopaque) was used. Which one of the following nursing measures would be included in his care?"

CHOICES

( X ) a.) Bed rest with bathroom privileges.

( X ) b.) Restricted fluid intake.

( X ) c.) Head of the bed elevated 45 degrees.

( O ) d.) Assessment of lower extremity movement and sensation.


RATIONALE: Neurologic status in the lower extremities is assessed frequently, as is the client's ability to void. This is done to determine if there is any nerve impairment. Regardless of the type of dye used for the test, bed rest is required for several hours after a myelogram. Fluid intake is encouraged to replace cerebrospinal fluid, to reduce headache, and to facilitate absorption of retained contrast media. Nursing care of the client after a myelogram depends in part on the type of dye used. For example, if an oil contrast such as Pantopaque was used, the client will usually lie flat for 8 to 12 hours. If a water-soluble contrast was used, the head of the bed is elevated 45 degrees for 8 to 24 hours. This position reduces the rate of upward dispersion of the contrast medium. NURSING PROCESS STEP: Implementation CLIENT NEEDS CATEGORY: Physiological integrity CLIENT NEEDS SUBCATEGORY: Reduction of risk potential
******************************

--> QUESTION NUMBER _ 942 _ about (MC)


QUESTION: "Which of the following categories of medications would the nurse anticipate being included in the conservative management of a client of with a herniated lumbar disk?"

CHOICES

( O ) a.) Muscle relaxants.

( X ) b.) Sedatives.

( X ) c.) Tranquilizers.

( X ) d.) Parenteral analgesics.


RATIONALE: Muscle relaxants and nonsteroidal anti-inflammatory drugs are frequently prescribed for the conservative management of herniated lumbar disks. In addition, the client may receive oral analgesics. Sedatives are not typically used in the conservative treatment of herniated lumbar disks. Tranquilizers are not typically used in the conservative treatment of herniated lumbar disks. Parenteral analgesics are not typically used in the conservative treatment of herniated lumbar disks. NURSING PROCESS STEP: Planning CLIENT NEEDS CATEGORY: Physiological integrity CLIENT NEEDS SUBCATEGORY: Pharmacological and parenteral therapies
******************************

--> QUESTION NUMBER _ 943 _ about (MC)


QUESTION: "The client with a lumbar laminectomy asks to be turned onto his side. It would be appropriate for the nurse to:"

CHOICES

( X ) a.) Ask the client to help by using an overhead trapeze to turn himself.

( X ) b.) Turn the client's shoulders first, followed by his hips and legs.

( X ) c.) Inform the client that because of his laminectomy, he may only lie supine.

( O ) d.) Get another nurse to help log roll the client into position.


RATIONALE: After a laminectomy, the client's spine must be maintained in proper alignment. The client who had a laminectomy may be turned to his side by logrolling him in one unit while keeping his back straight. It takes at least two people to perform this procedure correctly. Reaching up and using the trapeze will put stress on the operative are and cause the spinal column to twist. Such motions interfere with healing and can cause pain. Turning the shoulders then the hips will cause the spine to rotate which is contraindicated in the immediate postoperative period. Laminectomy clients should be assisted to side-lying positions to take pressure off the sacral area and stimulate circulation, however, position changes must be done so that the back stays in straight or neutral alignment. NURSING PROCESS STEP: Implementation CLIENT NEEDS CATEGORY: Safe, effective care environment CLIENT NEEDS SUBCATEGORY: Safety and infection control
******************************

--> QUESTION NUMBER _ 944 _ about (MC)


QUESTION: "To protect a client's skin under a back brace, the nurse should:"

CHOICES

( X ) a.) Place padding as necessary for a snug fit.

( O ) b.) Have the client wear a thin cotton shirt under the back brace.

( X ) c.) Lubricate the areas where the client's back brace will contact skin surfaces.

( X ) d.) Apply powder to the areas where the client's back brace will contact skin surfaces.


RATIONALE: Having the client wear a thin cotton shirt under a back brace helps to protect the skin and to keep the brace free of skin oils and perspiration. Using padding may increase pressure points. Lubricating the skin under the back brace will not provide the best protection from irritation by the brace. Powdering the skin under the back brace will not provide the best protection from irritation by the brace. NURSING PROCESS STEP: Implementation CLIENT NEEDS CATEGORY: Physiological integrity CLIENT NEEDS SUBCATEGORY: Reduction of risk potential
******************************

--> QUESTION NUMBER _ 945 _ about (MC)


QUESTION: "A client with a lumbar laminectomy ambulates for the first time after surgery and begins to feel faint. Which nursing action would be best until help arrives?"

CHOICES

( X ) a.) Have the client close his eyes for a few minutes.

( X ) b.) Maneuver the client to a sitting position on the floor.

( O ) c.) Separate her or his feet to form a wide base of support and have the client rest against the nurse's hip.

( X ) d.) Have the client separate his feet to form a wide base of support then bend at the waist to place his head near his knees.


RATIONALE: A client who feels faint while walking with the nurse should rest on the nurse's hip. This maneuver is relatively easy and can be maintained until help is available. Having the client close his eyes is unlikely to relieve symptoms of fainting. Maneuvering the client to the floor requires considerable strength and may injure the client, especially when done quickly. This client should not bend at the waist because of his recent back surgery. NURSING PROCESS STEP: Implementation CLIENT NEEDS CATEGORY: Physiological integrity CLIENT NEEDS SUBCATEGORY: Physiological adaptation
******************************

--> QUESTION NUMBER _ 946 _ about (MC)


QUESTION: "Which of the following instructions regarding body mechanics would be most appropriate for helping the client to avoid back injury?"

CHOICES

( X ) a.) Pull objects rather than pushing them.

( X ) b.) Sleep on a soft mattress.

( O ) c.) Avoid prolonged sitting and standing.

( X ) d.) Sit in chairs with soft cushions.


RATIONALE: Prolonged sitting and standing should be avoided because they strain the lower back. Pushing objects rather than pulling them will help decrease back strain. Clients should select a semi-firm to firm mattress to provide back support. When sitting, the client should choose a chair with good support and a straight back. The client should sit with feet flat on the floor. NURSING PROCESS STEP: Implementation CLIENT NEEDS CATEGORY: Physiological integrity CLIENT NEEDS SUBCATEGORY: Basic care and comfort
******************************

--> QUESTION NUMBER _ 947 _ about (MC)


QUESTION: "Which of the following symptoms is not typically associated with peripheral arterial disease?"

CHOICES

( O ) a.) Ankle edema.

( X ) b.) Intermittent claudication.

( X ) c.) Decreased or absent pulses.

( X ) d.) Cool skin.


RATIONALE: Ankle edema is associated with venous insufficiency and stasis. Inadequate arterial circulation produces symptoms of hypoxia; exercise causes moderate to severe cramping pain (intermittent claudication). The progressive occlusion of the arteries weakens the pulses, making them difficult or impossible to palpate. Because blood flow is decreased, the skin is cool to the touch. NURSING PROCESS STEP: Assessment CLIENT NEEDS CATEGORY: Physiological integrity CLIENT NEEDS SUBCATEGORY: Physiological adaptation
******************************

--> QUESTION NUMBER _ 948 _ about (MC)


QUESTION: "To assess the client's dorsalis pedis pulse, the nurse should palpate the:"

CHOICES

( X ) a.) Medial surface of the ankle.

( X ) b.) Lateral surface of the ankle.

( X ) c.) Ventral aspect of the top of the foot.

( O ) d.) Medial aspect of the dorsum of the foot.


RATIONALE: The dorsalis pedis pulse is found on the medial aspect of the dorsal surface of the foot in line with the big toe. The posterior tibial pulse is on the medial surface of the ankle just behind the medial malleolus. The dorsalis pedis pulse is found on the medial aspect of the dorsal surface of the foot in line with the big toe. The dorsalis pedis pulse is found on the medial aspect of the dorsal surface of the foot in line with the big toe. NURSING PROCESS STEP: Assessment CLIENT NEEDS CATEGORY: Health promotion and maintenance CLIENT NEEDS SUBCATEGORY: None
******************************

--> QUESTION NUMBER _ 949 _ about (MC)


QUESTION: "Which of the following clinical manifestations would be most indicative of complete arterial obstruction in the lower extremities?"

CHOICES

( X ) a.) Aching pain.

( X ) b.) Burning sensations.

( X ) c.) Numbness and tingling.

( O ) d.) Coldness.


RATIONALE: Coldness is the assessment finding most consistent with complete arterial obstruction. Other expected findings would include paralysis and pallor. Aching pain, an earlier sign of tissue hypoxia and ischemia, is associated with incomplete obstruction. Burning sensations are earlier signs of tissue hypoxia and ischemia and are associated with incomplete obstruction. Numbness and tingling, which are earlier signs of tissue hypoxia and ischemia, are associated with incomplete obstruction. NURSING PROCESS STEP: Assessment CLIENT NEEDS CATEGORY: Physiological integrity CLIENT NEEDS SUBCATEGORY: Physiological adaptation
******************************

--> QUESTION NUMBER _ 950 _ about (MC)


QUESTION: "The client returns from surgery for a below-the-knee amputation with the residual limb covered with dressings and a woven elastic bandage. At first, the bandage was dry. Now, 30 minutes later, the nurse notices a small amount of bloody drainage. What should be the priority action?"

CHOICES

( X ) a.) Notify the physician.

( O ) b.) Mark the area of drainage.

( X ) c.) Change the dressing.

( X ) d.) Reinforce the dressing.


RATIONALE: The nurse should mark the bloody drainage and observe it again in 10 minutes to assess if the bleeding is continuing. There is no need to notify the physician immediately because some oozing and bloody drainage are expected. A fresh postoperative dressing should not be changed unless the physician orders it. Although the wound edges will be closed, no epithelialization has occurred yet to protect the deep tissues. Undressing the wound at this point increases the risk of a wound infection. Given the slight amount of drainage, there is no need to reinforce the dressing. NURSING PROCESS STEP: Implementation CLIENT NEEDS CATEGORY: Physiological integrity CLIENT NEEDS SUBCATEGORY: Physiological adaptation
******************************

--> QUESTION NUMBER _ 951 _ about (MC)


QUESTION: "What would be the most important nursing intervention in caring for the client's residual limb during the first 24 hours after amputation of the left leg?"

CHOICES

( X ) a.) Keeping the residual limb flat on the bed.

( X ) b.) Abducting the residual limb on a scheduled basis.

( X ) c.) Applying traction to the residual limb.

( O ) d.) Elevating the residual limb on a pillow.


RATIONALE: Elevating the residual limb on a pillow for the first 24 hours after surgery helps prevent edema and promotes comfort by increasing venous return. Elevating the residual limb for longer than the first 24 hours is contraindicated because of the potential for developing a hip flexion contracture. Keeping the limb flat will be an important intervention after the first 24 hours. Preventing excessive swelling, however, is a priority in the first 24 hours. Adducting the residual limb on a scheduled basis prevents abduction contracture. Traction may be used to prevent or treat a hip flexion contracture, however, not in the first 24 hours. NURSING PROCESS STEP: Implementation CLIENT NEEDS CATEGORY: Physiological integrity CLIENT NEEDS SUBCATEGORY: Physiological adaptation
******************************

--> QUESTION NUMBER _ 952 _ about (MC)


QUESTION: "Which of the following nursing goals would take priority when planning for the client's physical mobility after amputation?"

CHOICES

( O ) a.) Preventing contractures.

( X ) b.) Promoting comfort.

( X ) c.) Preventing edema.

( X ) d.) Preventing phantom-limb pain.


RATIONALE: Preventing joint contractures is essential to physical mobility. Promoting comfort is an appropriate immediate postoperative nursing goal, but attaining it does not affect physical mobility in the extended postoperative periods. Preventing edema is an appropriate immediate postoperative nursing goal, but attaining it does not affect physical mobility in the immediate and extended postoperative periods. Phantom-limb pain begins 2 weeks to 2 months after amputation. It occurs briefly in about 30% of clients, but only about 2% experience persistent pain. NURSING PROCESS STEP: Planning CLIENT NEEDS CATEGORY: Physiological integrity CLIENT NEEDS SUBCATEGORY: Physiological adaptation
******************************

--> QUESTION NUMBER _ 953 _ about (MC)


QUESTION: "The second morning after surgery for a below-the-knee amputation of the left leg, the client says, "This sounds crazy, but I feel my left toes tingling." This statement would indicate to the nurse that he is experiencing a:"

CHOICES

( X ) a.) Denial reaction.

( O ) b.) Phantom-limb sensation.

( X ) c.) Hallucination.

( X ) d.) Body image disturbance.


RATIONALE: Descriptions of sensations, painful and otherwise, in the amputated part are common and are known as phantom-limb sensations. The client should be reassured that these sensations are normal and are not a sign of a mental problem. Denial may be present after amputation; signs include refusal to look at or talk about the amputation. The client is not hallucinating. Phantom limb pain is a well-documented sensation and not associated with a psychiatric disorders. Body image disturbances can develop after amputation due to fear, grief, loss of locomotion, and decreased self-esteem related to the loss of the body part; however, phantom limb pain is a very real sensation and should not be treated as a psychosocial phenomenon. NURSING PROCESS STEP: Analysis CLIENT NEEDS CATEGORY: Physiological integrity CLIENT NEEDS SUBCATEGORY: Physiological adaptation
******************************

--> QUESTION NUMBER _ 954 _ about (MC)


QUESTION: "The client with an above-the-knee amputation (AKA) is to be fitted with a functioning prosthesis. The nurse has been teaching him how to care for his residual limb. Which behavior would demonstrate that the client has an understanding of proper residual limb care? The client:"

CHOICES

( X ) a.) Applies powder to the residual limb.

( X ) b.) Inspects the residual limb weekly with a mirror.

( X ) c.) Removes the prosthesis whenever he sits down.

( O ) d.) Washes and dries the residual limb daily.


RATIONALE: Washing and thoroughly drying the residual limb daily are important hygiene measures to prevent infection. Nothing should be applied to the residual limb after it is cleansed. Powder may cause excessive drying and cracking of the skin, and cream may soften the skin excessively. The residual limb should be inspected daily with a mirror for early signs of skin breakdown. To reduce residual limb swelling, the prosthesis should be removed only at night. NURSING PROCESS STEP: Evaluation CLIENT NEEDS CATEGORY: Physiological integrity CLIENT NEEDS SUBCATEGORY: Reduction of risk potential
******************************

--> QUESTION NUMBER _ 955 _ about (MC)


QUESTION: "The nurse has been instructing the client about how to prepare meals that are low in fat, cholesterol, and sodium. Which of these comments would indicate that he needs additional teaching?"

CHOICES

( X ) a.) "I'll eat water-packed tuna."

( X ) b.) "I'll use a Teflon-coated pan when cooking."

( O ) c.) "I'll eat more liver with onions."

( X ) d.) "I'll avoid using steak sauce and catsup."


RATIONALE: Liver and organ meats are high in cholesterol and saturated fat and should be limited. Water-packed tuna is one of the leanest fish available. Using a Teflon-coated pan when cooking reduces the need for shortening. Steak sauce and catsup are high in sodium. NURSING PROCESS STEP: Evaluation CLIENT NEEDS CATEGORY: Physiological integrity CLIENT NEEDS SUBCATEGORY: Basic care and comfort
******************************

--> QUESTION NUMBER _ 956 _ about (MC)


QUESTION: "A client is admitted to the orthopedic unit in balanced skeletal traction using a Thomas splint and Pearson attachment. The primary purpose of traction in this case is to:"

CHOICES

( X ) a.) Prevent neurologic damage.

( O ) b.) Realign fracture fragments.

( X ) c.) Control internal bleeding.

( X ) d.) Maintain skin integrity.


RATIONALE: Traction promotes realignment of the bone fragments. This will facilitate subsequent internal fixation. Traction immobilizes the fracture site and may increase the client's comfort. Mobilization could result in further damage. The use of traction does not prevent a neurologic damage and can, in fact, cause pressure that leads to nerve damage. Traction increases circulation to the affected part but does not control internal bleeding. Traction may create, rather than prevent, a problem with skin integrity. NURSING PROCESS STEP: Planning CLIENT NEEDS CATEGORY: Physiological integrity CLIENT NEEDS SUBCATEGORY: Reduction of risk potential
******************************

--> QUESTION NUMBER _ 957 _ about (MC)


QUESTION: "The nurse is responsible for maintaining effective traction. Which of the following conditions is necessary for effective traction?"

CHOICES

( X ) a.) The weights rest securely on the bed frame.

( O ) b.) The ropes are in the wheel grooves of the pulleys.

( X ) c.) The client is positioned low in the bed.

( X ) d.) The weights are increased by one-half pound each shift.


RATIONALE: For the weights to maintain the therapeutic effect of the traction, they must be properly positioned, free hanging, and should be removed only in life-threatening situations. Effective traction depends on the client being positioned at the head of the bed. Sufficient weight is applied initially to overcome spasm in affected muscles. As the muscles relax, the weight may be reduced. Weights must hang freely and not be resting on anything. The client must be positioned at the head of the bed for the traction to be effective and the weights to hang freely. The amount of weight used is determined by the physician and is not changed each shift. NURSING PROCESS STEP: Implementation CLIENT NEEDS CATEGORY: Physiological integrity CLIENT NEEDS SUBCATEGORY: Reduction of risk potential
******************************

--> QUESTION NUMBER _ 958 _ about (MC)


QUESTION: "When a client is placed in balanced skeletal traction, which of the following nursing actions would be appropriate?"

CHOICES

( O ) a.) Ensuring that the traction weights hang freely from the bed at all times.

( X ) b.) Increasing the traction weight gradually as the client's tolerance increases.

( X ) c.) Applying and removing the traction weights at regular intervals throughout the day.

( X ) d.) Removing the weights briefly as necessary to reposition the client in bed.


RATIONALE: In balanced skeletal traction, the appropriate pressures and counter pressures are applied to the fracture site, with the traction weights hanging freely at all times. The amount of traction weight used is determined by radiography and the alignment of the fracture. These weights are in place continuously and should never be lifted, reduced, or eliminated. These weights are in place continuously and should never be lifted, reduced, or eliminated. NURSING PROCESS STEP: Implementation CLIENT NEEDS CATEGORY: Physiological integrity CLIENT NEEDS SUBCATEGORY: Reduction of risk potential
******************************

--> QUESTION NUMBER _ 959 _ about (MC)


QUESTION: "Which of the following treatments would most likely be used to treat the client if fat emboli developed?"

CHOICES

( X ) a.) Hypothermia.

( O ) b.) Supplemental oxygen.

( X ) c.) Intravenous heparin.

( X ) d.) Cholesterol-lowering drugs.


RATIONALE: Respiratory failure and death are associated with fat emboli. Respiratory support is provided by administering high concentrations of oxygen. Oxygen therapy appears to reduce the surface tension of the fat globules and supports respiratory function by reducing hypoxia. Hypothermia has no place in the treatment of fat emboli. Intravenous heparin is the treatment for thrombotic emboli. Cholesterol-lowering drugs act in the liver to reduce cholesterol production or bind with cholesterol in the GI tract. They will have no effect on a fat embolus. NURSING PROCESS STEP: Planning CLIENT NEEDS CATEGORY: Physiological integrity CLIENT NEEDS SUBCATEGORY: Physiological adaptation
******************************

--> QUESTION NUMBER _ 960 _ about (MC)


QUESTION: "To prevent infection and osteomyelitis, the nurse provides pin site care and inspects the site daily for evidence of infection. Which of the following clinical manifestations at the pin site would alert the nurse to infection?"

CHOICES

( X ) a.) Slight serous oozing.

( X ) b.) Lack of scab formation.

( X ) c.) Itching.

( O ) d.) Pain.


RATIONALE: Inflammation, evidenced by pain, swelling, and redness, is one of the early signs of infection and needs prompt intervention. Slight oozing at the pin site is expected and decreases bacteria in the pin tract. Crusting or scab formation should be prevented because either may trap bacteria in the pin tract. Itching at the pin site may be due to dryness or irritation. NURSING PROCESS STEP: Assessment CLIENT NEEDS CATEGORY: Physiological integrity CLIENT NEEDS SUBCATEGORY: Reduction of risk potential
******************************

--> QUESTION NUMBER _ 961 _ about (MC)


QUESTION: "The client has a nursing diagnosis of Constipation related to decreased mobility secondary to traction. A care plan that incorporates which of the following breakfasts would be most helpful in reestablishing a normal bowel routine?"

CHOICES

( X ) a.) Eggs and bacon, buttered white toast, orange juice, and coffee.

( X ) b.) Corn flakes with sliced banana, milk, and English muffin and jelly.

( X ) c.) Orange juice, breakfast pastries (doughnut and Danish), and coffee.

( O ) d.) An orange, raisin bran and milk, and wheat toast with butter.


RATIONALE: High-fiber foods provide bulk and decrease water absorption in the bowel. Whole grains and fruits (not juices which often are strained) are recommended. Proteins, white bread, and liquids contain very little fiber. This choice is higher in fiber than the others, but not high enough to enhance bowel elimination. Processed foods and breads contain little fiber. NURSING PROCESS STEP: Planning CLIENT NEEDS CATEGORY: Physiological integrity CLIENT NEEDS SUBCATEGORY: Physiological adaptation
******************************

--> QUESTION NUMBER _ 962 _ about (MC)


QUESTION: "A client has suffered a C7 spinal cord injury. Which of the following would be the most important nursing intervention during the acute stage of her injury?"

CHOICES

( X ) a.) Turning and repositioning every 2 hours.

( X ) b.) Maintaining proper alignment.

( O ) c.) Maintaining a patent airway.

( X ) d.) Monitoring vital signs.


RATIONALE: Initial care is focused on establishing and maintaining a patent airway and supporting ventilation. Innervation to the intercostal muscles is affected; if spinal edema extends to the C4 level, paralysis of the diaphragm usually occurs. The effects and extent of edema are unpredictable in the first hours, and respiratory status must be closely monitored. Suction equipment should be readily available. Turning and positioning is important, but the priority nursing intervention is maintaining a patent airway. Maintaining proper alignment is important, but the priority nursing intervention is maintaining a patent airway. Monitoring vital signs is important, but the priority nursing intervention is maintaining a patent airway. NURSING PROCESS STEP: Implementation CLIENT NEEDS CATEGORY: Physiological integrity CLIENT NEEDS SUBCATEGORY: Physiological adaptation
******************************

--> QUESTION NUMBER _ 963 _ about (MC)


QUESTION: "The nurse recognizes that spinal shock is likely to persist for the first several weeks after the injury. Which of the following symptoms would be unexpected during the period of spinal shock?"

CHOICES

( X ) a.) Tachycardia.

( X ) b.) Rapid respirations.

( O ) c.) Hypertension.

( X ) d.) Dry, warm skin.


RATIONALE: Hypertension would not be expected. Spinal shock produces massive vasodilation and subsequent pooling of blood in the peripheral circulation. The client is relatively hypovolemic and exhibits tachycardia. The client is typically tachypneic. The client often exhibits flushed but dry skin. NURSING PROCESS STEP: Assessment CLIENT NEEDS CATEGORY: Physiological integrity CLIENT NEEDS SUBCATEGORY: Physiological adaptation
******************************

--> QUESTION NUMBER _ 964 _ about (MC)


QUESTION: "Passive range-of-motion exercises for the legs and assisted range-of-motion exercises for the arms are part of a client's care regimen. Which observation by the nurse would indicate a successful outcome of this treatment?"

CHOICES

( O ) a.) Free, easy movement of the joints.

( X ) b.) Absence of paralytic foot-drop.

( X ) c.) External rotation of the hips at rest.

( X ) d.) Absence of tissue ischemia over bony prominences.


RATIONALE: Range-of-motion exercises help preserve joint motion and stimulate circulation. Contractures develop rapidly in clients with spinal cord injuries, and the absence of this complication indicates treatment success. Range of motion will keep the ankle joints freely mobile. Foot-drop, however, is prevented by proper positioning of the ankle and foot which is usually accomplished with high top sneakers or splints. External rotation of the hips is prevented by using trochanter rolls. Local ischemia over bony prominences is prevented by following a regular turning schedule. NURSING PROCESS STEP: Evaluation CLIENT NEEDS CATEGORY: Physiological integrity CLIENT NEEDS SUBCATEGORY: Physiological adaptation
******************************

--> QUESTION NUMBER _ 965 _ about (MC)


QUESTION: "The client's cervical fracture is surgically repaired. Once healing has begun, daily physical therapy sessions are scheduled that include using a tilt table. After the therapist places the client at a 45-degree angle, the nurse should monitor the client for which of the following?"

CHOICES

( X ) a.) Hypertension.

( X ) b.) Pedal edema.

( X ) c.) Facial flushing.

( O ) d.) Dizziness.


RATIONALE: Lack of vasomotor tone in the lower extremities causes venous pooling, and the client may become hypotensive and dizzy when positioned upright. The tilt table is used to help the client overcome vasomotor instability and tolerate an upright position. The client's blood pressure is likely to drop, not increase. Some pedal edema could occur, but it would develop gradually and be less problematic than the hypotension. Elastic stockings are sometimes used to facilitate venous return from the legs. Signs and symptoms of insufficient cerebral circulation are pallor, diaphoresis, tachycardia, and nausea. NURSING PROCESS STEP: Assessment CLIENT NEEDS CATEGORY: Physiological integrity CLIENT NEEDS SUBCATEGORY: Physiological adaptation
******************************

--> QUESTION NUMBER _ 966 _ about (MC)


QUESTION: "The nurse realizes that the client is at risk for autonomic dysreflexia. Which of the following symptoms would indicate this condition?"

CHOICES

( O ) a.) Sudden, severe hypertension.

( X ) b.) Bradycardia.

( X ) c.) Paralytic ileus.

( X ) d.) Hot, dry skin.


RATIONALE: With a cervical injury, the client has sympathetic fibers that can be stimulated to fire reflexively. The firing is cut off from brain control and is both reflexive and massive. It classically produces pounding headache and dangerously elevated blood pressure, "goose bumps," and profuse sweating. Bradycardia typically occurs during spinal shock, not during autonomic dysreflexia. Paralytic ileus typically occurs during spinal shock, not during autonomic dysreflexia. Hot and dry skin and typically occurs during spinal shock, not during autonomic dysreflexia. NURSING PROCESS STEP: Assessment CLIENT NEEDS CATEGORY: Physiological integrity CLIENT NEEDS SUBCATEGORY: Physiological adaptation
******************************

--> QUESTION NUMBER _ 967 _ about (MC)


QUESTION: "If autonomic dysreflexia occurs, what would be the priority nursing intervention?"

CHOICES

( X ) a.) Administer nitroprusside sodium (Nipride) intravenously.

( X ) b.) Call the physician.

( O ) c.) Place the client in Fowler's position.

( X ) d.) Send a urine sample for culture.


RATIONALE: Autonomic dysreflexia is a medical emergency. The rising blood pressure can cause cerebrovascular accident, blindness, or even death. Placing the client in Fowler's position lowers blood pressure. Administering nitroprusside sodium intravenously is appropriate if the conservative measures are ineffective. Although notifying the physician is important, it is more essential that the nurse intervene immediately in the situation. A urine sample for culture should be obtained if the client has an elevated temperature and no other cause for the dysreflexia is found. A urinary tract infection may be causing symptoms. NURSING PROCESS STEP: Implementation CLIENT NEEDS CATEGORY: Physiological integrity CLIENT NEEDS SUBCATEGORY: Physiological adaptation
******************************

--> QUESTION NUMBER _ 968 _ about (MC)


QUESTION: "The nurse assesses the client to determine the cause of autonomic dysreflexia. The nurse would prioritize assessment based on the knowledge that the most common stimulus for an autonomic dysreflexia episode is:"

CHOICES

( X ) a.) Bowel distention.

( O ) b.) Bladder distention.

( X ) c.) Anxiety.

( X ) d.) Rising intracranial pressure.


RATIONALE: The dysreflexia occurs from a sympathetic response to autonomic nervous system stimulation. A distended bladder is the most common cause. After placing the client in Fowler's position, the nurse should check the urinary catheter for patency. Bowel distention may also trigger the syndrome. However, the most common cause of autonomic dysreflexia is bladder distention, so the nurse should assess for this first. Anxiety does not cause autonomic dysreflexia. Increased intracranial pressure does not cause autonomic dysreflexia. NURSING PROCESS STEP: Assessment CLIENT NEEDS CATEGORY: Physiological integrity CLIENT NEEDS SUBCATEGORY: Physiological adaptation
******************************

--> QUESTION NUMBER _ 969 _ about (MC)


QUESTION: "Urinary tract infection is a serious problem after spinal cord injury. Which of the following interventions help prevent this?"

CHOICES

( X ) a.) Drinking a glass of citrus fruit juice at every meal.

( O ) b.) Drinking at least 2000 mL of fluid daily.

( X ) c.) Adding extra protein to the daily diet.

( X ) d.) Ensuring that the urine remains alkaline.


RATIONALE: As soon as the client's vasomotor status stabilizes and she is not susceptible to fluid volume overload, it is essential that she drink at least 2000 mL of fluid daily, unless contraindicated by her medical condition. Increased fluid intake helps flush out bacteria and prevents urinary stasis. Citrus juices are not encouraged. They can promote a urinary tract infection because they are alkaline. Most citrus fruits are not metabolized as acids in the body. Extra protein does not decrease the potential for a urinary tract infection. Ingesting an acid-ash diet forms acid urine which helps prevent urinary tract infection. NURSING PROCESS STEP: Implementation CLIENT NEEDS CATEGORY: Physiological integrity CLIENT NEEDS SUBCATEGORY: Basic care and comfort
******************************

--> QUESTION NUMBER _ 970 _ about (MC)


QUESTION: "Before cataract surgery, the nurse is to instill several types of eye drops into a client's right eye. The accepted abbreviation for the right eye is:"

CHOICES

( O ) a.) OD.

( X ) b.) OS.

( X ) c.) OU.

( X ) d.) RE.


RATIONALE: The accepted abbreviation for the right eye is OD, which stands for oculus dexter. OS (oculus sinister) refers to the left eye. OU (oculus uterque) refers to both eyes. RE is not an accepted abbreviation for the right eye. NURSING PROCESS STEP: Implementation CLIENT NEEDS CATEGORY: Physiological integrity CLIENT NEEDS SUBCATEGORY: Pharmacological and parenteral therapies
******************************

--> QUESTION NUMBER _ 971 _ about (MC)


QUESTION: "Which of the following statements indicates the client has understood the instructions to follow at home after cataract surgery?"

CHOICES

( X ) a.) "I may not watch television for 3 weeks."

( X ) b.) "I should keep my protective eye shield in place at all times."

( O ) c.) "I should not bend over to pick up objects from the floor."

( X ) d.) "I can lift what I want."


RATIONALE: When picking up objects from the floor, the client should be instructed to bend at the knees and keep the head up and back straight. The client may watch television and read in moderation. The eye shield is usually worn only at night. Lifting should be restricted for the first week to less than 15 pounds. NURSING PROCESS STEP: Evaluation CLIENT NEEDS CATEGORY: Physiological integrity CLIENT NEEDS SUBCATEGORY: Basic care and comfort
******************************

--> QUESTION NUMBER _ 972 _ about (MC)


QUESTION: "An essential aspect of the plan of care for the client after cataract removal surgery would be to:"

CHOICES

( X ) a.) Increase cardiac output.

( X ) b.) Prevent fluid volume excess.

( X ) c.) Maintain a darkened environment.

( O ) d.) Promote safety at home.


RATIONALE: Promoting safety is a priority goal for this client. The client's vision will not be clear, and she may need to wear an eye patch after surgery. Orienting the client to the physical environment, assisting her during ambulation, and following other safety precautions to reduce the risk of injury are required. Cardiac output has no relationship to cataract surgery. Fluid volume excess has no relationship to cataract surgery. Maintaining a darkened environment is neither necessary nor safe. NURSING PROCESS STEP: Planning CLIENT NEEDS CATEGORY: Safe, effective care environment CLIENT NEEDS SUBCATEGORY: Safety and infection control
******************************

--> QUESTION NUMBER _ 973 _ about (MC)


QUESTION: "What information about vision would be most important for the nurse to include in the discharge plan of a client who had cataract removal?"

CHOICES

( X ) a.) "You will need to wear corrective glasses or contact lenses."

( X ) b.) "You will need to wear glasses only until the eye heals."

( X ) c.) "You will need to wear cataract glasses that correct vision by magnifying objects."

( O ) d.) "You will need to relearn to judge distances accurately."


RATIONALE: Even with glasses, the client who has had cataract surgery may have changes in depth perception. The client may need to relearn to judge distances accurately to walk safely. The client will need glasses or contact lenses to correct visual acuity, however, depth perception may vary. The client will need glasses or contact lenses to correct visual acuity. There is no need to wear glasses that magnify objects. NURSING PROCESS STEP: Implementation CLIENT NEEDS CATEGORY: Physiological integrity CLIENT NEEDS SUBCATEGORY: Basic care and comfort
******************************

--> QUESTION NUMBER _ 974 _ about (MC)


QUESTION: "After cataract removal surgery, the nurse teaches the client about activities that she can do at home. Which of the following activities would be contraindicated?"

CHOICES

( X ) a.) Walking down the hall unassisted.

( X ) b.) Lying in bed on the nonoperative side.

( X ) c.) Performing isometric exercises.

( O ) d.) Bending over the sink to wash her hair.


RATIONALE: Bending over the sink to wash hair is contraindicated after cataract surgery because it increases intraocular pressure. The client should be taught to tilt her head backward slightly when washing her hair. Walking is not contraindicated. Lying in bed on the nonoperative side is not contraindicated. Performing isometric exercises is not contraindicated. NURSING PROCESS STEP: Implementation CLIENT NEEDS CATEGORY: Physiological integrity CLIENT NEEDS SUBCATEGORY: Basic care and comfort
******************************

--> QUESTION NUMBER _ 975 _ about (MC)


QUESTION: "An anticipated outcome for the client after cataract removal surgery would include which of the following? "

CHOICES

( X ) a.) The client states her vision is clear.

( X ) b.) The client states her infection is under control.

( O ) c.) The client describes methods to prevent an increase in intraocular pressure.

( X ) d.) The client states she is able to administer parenteral pain medication.


RATIONALE: Preventing an increase in intraocular pressure is the primary concern after cataract removal. Vision will remain unclear temporarily after surgery. Infection, although it may occur, is not an anticipated outcome of the surgery. Parenteral pain medication at home is not required. NURSING PROCESS STEP: Evaluation CLIENT NEEDS CATEGORY: Physiological integrity CLIENT NEEDS SUBCATEGORY: Physiological adaptation
******************************

--> QUESTION NUMBER _ 976 _ about (MC)


QUESTION: "The client with a retinal detachment does not understand what happened to his eye. Which of the following explanations by the nurse would most accurately describe the pathology of retinal detachment?"

CHOICES

( X ) a.) "A tear in the retina permits the escape of vitreous humor from the eye."

( X ) b.) "The optic nerve is damaged when it is exposed to vitreous humor."

( O ) c.) "The two layers of the retina separate, allowing fluid to enter between them."

( X ) d.) "Retinal injury produces inflammation and edema that increase intraocular pressure."


RATIONALE: In retinal detachment, the two layers of the retina separate as a result of a small hole or tear, trauma, or degeneration. Vitreous humor seeps into the tear and separates the retinal layers. Vitreous humor does not leak out of the eye or cause any direct damage to the optic nerve. Vitreous humor does not cause any direct damage to the optic nerve. Increased intraocular pressure is not associated with retinal detachment. NURSING PROCESS STEP: Implementation CLIENT NEEDS CATEGORY: Physiological integrity CLIENT NEEDS SUBCATEGORY: Physiological adaptation
******************************

--> QUESTION NUMBER _ 977 _ about (MC)


QUESTION: "The nurse should instruct the client to do which of the following activities in the postoperative period after scleral buckling?"

CHOICES

( X ) a.) Encouraging deep breathing and coughing every 2 hours.

( O ) b.) Assessing for eye drainage.

( X ) c.) Applying pressure dressings to both eyes.

( X ) d.) Enforcing strict bed rest.


RATIONALE: After eye surgery, the client should be taught to assess for excessive or purulent drainage that may indicate infection. Coughing should be avoided as this increases intraocular pressure. Pressure dressings are not applied to the eyes after surgery, although general eye patching may be used temporarily. Activity may vary but usually does not require strict bed rest. NURSING PROCESS STEP: Implementation CLIENT NEEDS CATEGORY: Physiological integrity CLIENT NEEDS SUBCATEGORY: Physiological adaptation
******************************

--> QUESTION NUMBER _ 978 _ about (MC)


QUESTION: "Which of the following clinical manifestations commonly occur in retinal detachment?"

CHOICES

( X ) a.) Sudden severe eye pain and colored halos around lights.

( X ) b.) Inability to move the eye and loss of light accommodation.

( X ) c.) A tearing sensation and increased lacrimation.

( O ) d.) Flashing lights and visual field loss.


RATIONALE: A client with retinal detachment frequently reports flashing lights in the affected eye followed by a loss of vision commonly described as a curtain being slowly drawn across the eye. The detachment is painless, and the client will not report pain. Retinal detachment does not involve the eye muscles. Retinal detachment does not cause lacrimation. NURSING PROCESS STEP: Assessment CLIENT NEEDS CATEGORY: Physiological integrity CLIENT NEEDS SUBCATEGORY: Physiological adaptation
******************************

--> QUESTION NUMBER _ 979 _ about (MC)


QUESTION: "Before the surgical repair of a detached retina, the client is placed on flat bed rest. This position :"

CHOICES

( X ) a.) Helps reduce intraocular pressure.

( X ) b.) Facilitates drainage from the eye.

( X ) c.) Keeps the client safe while confined to bed.

( O ) d.) Helps prevent further retinal detachment or tearing.


RATIONALE: The client's position is determined by the location of the retinal tear. The rationale for rest is the hope that the retina will fall back into place as much as possible before surgery, which will facilitate adherence of the retina to the choroid. Increased intraocular pressure is not a problem in retinal detachment. There should be no external drainage from the eye. The client is placed on bed rest to facilitate treatment of the eye, not to keep the client safe. NURSING PROCESS STEP: Implementation CLIENT NEEDS CATEGORY: Physiological integrity CLIENT NEEDS SUBCATEGORY: Physiological adaptation
******************************

--> QUESTION NUMBER _ 980 _ about (MC)


QUESTION: "In discharge planning after scleral buckling, the nurse should ensure that the client understands the need for initial activity restriction at home. Which of the following activities would be contraindicated during the early recovery period?"

CHOICES

( X ) a.) Watching television.

( O ) b.) Reading.

( X ) c.) Talking on the telephone.

( X ) d.) Walking in the yard.


RATIONALE: Although restful, reading involves too much jerky eye movement and should be avoided during recovery. Watching television is an appropriate activity and can be encouraged. Talking on the phone is an appropriate activity . Walking outdoors is an appropriate activity and can be encouraged. NURSING PROCESS STEP: Planning CLIENT NEEDS CATEGORY: Physiological integrity CLIENT NEEDS SUBCATEGORY: Physiological adaptation
******************************

--> QUESTION NUMBER _ 981 _ about (MC)


QUESTION: "The nurse would evaluate that the client understands his home care instructions after scleral buckling for a detached retina if he says his activity should include:"

CHOICES

( O ) a.) Avoiding abrupt movements of the head.

( X ) b.) Exercising the eye muscles each day.

( X ) c.) Turning the entire head rather than just the eyes for sight.

( X ) d.) Avoiding activities requiring good depth perception.


RATIONALE: During recovery, the client should be instructed to avoid abrupt or jarring head movements. Activities such as shampooing or brushing hair may be restricted. No specific eye exercises are prescribed. There is no need to turn the head to see instead of the eyes. Depth perception is not specially affected by this surgery. NURSING PROCESS STEP: Evaluation CLIENT NEEDS CATEGORY: Physiological integrity CLIENT NEEDS SUBCATEGORY: Physiological adaptation
******************************

--> QUESTION NUMBER _ 982 _ about (MC)


QUESTION: "The nurse evaluates the client's ability to instill eye drops correctly. The client correctly demonstrates the procedure when he:"

CHOICES

( X ) a.) Blows his nose immediately after administering the eye drops.

( X ) b.) Positions himself on his right side to instill the eye drops.

( O ) c.) Instills the eye drops into the conjunctival sac.

( X ) d.) Wipes the tip of the eye drop applicator with a disposable tissue.


RATIONALE: Proper technique for instilling eye drops includes instilling the eye drops in the conjunctival sac. There is no need for the client to blow his nose after eye drop administration. Proper technique for instilling eye drops includes being in a supine position. The applicator tip should remain sterile. NURSING PROCESS STEP: Evaluation CLIENT NEEDS CATEGORY: Physiological integrity CLIENT NEEDS SUBCATEGORY: Pharmacological and parenteral therapies
******************************

--> QUESTION NUMBER _ 983 _ about (MC)


QUESTION: "The nurse would plan to teach the client to administer which of the following drugs for open-angle glaucoma?"

CHOICES

( O ) a.) Pilocarpine hydrochloride.

( X ) b.) Atropine sulfate.

( X ) c.) Scopolamine hydrobromide.

( X ) d.) Acetazolamide (Diamox).


RATIONALE: Pilocarpine hydrochloride is a commonly prescribed miotic that causes pupillary constriction and produces negligible systemic effects. Atropine sulfate has a mydriatic effect causing pupillary dilation. Scopolamine hydrobromide has a mydriatic effect causing pupillary dilation. Acetazolamide, a carbonic anhydrase inhibitor, decreases secretion of aqueous humor in the eye, thus lowering intraocular pressure. This drug is prescribed for acute angle-closure glaucoma. NURSING PROCESS STEP: Planning CLIENT NEEDS CATEGORY: Physiological integrity CLIENT NEEDS SUBCATEGORY: Pharmacological and parenteral therapies
******************************

--> QUESTION NUMBER _ 984 _ about (MC)


QUESTION: "The most effective health-promotion measure related to glaucoma that the nurse could teach clients is:"

CHOICES

( X ) a.) Prompt treatment of all eye infections.

( X ) b.) Avoidance of extended-wear contact lenses by older people.

( O ) c.) Annual intraocular pressure measurements for people older than 40 years.

( X ) d.) Appropriate blood pressure control.


RATIONALE: The most effective health-promotion measure associated with glaucoma is annual intraocular pressure measurements after 40 years of age. People who are at risk for developing glaucoma, such as those with diabetes or hypertension, African Americans, and people with a family history of glaucoma, should have their intraocular pressure checked annually after 35 years of age. Glaucoma is insidious, basically asymptomatic, and must be diagnosed before the client becomes aware of any vision changes. Eye infections should be treated promptly, however, they are not a risk factor for glaucoma. Contact lens use is not associated with glaucoma. Blood pressure control has multiple health benefits but is not implicated in glaucoma. NURSING PROCESS STEP: Implementation CLIENT NEEDS CATEGORY: Health promotion and maintenance CLIENT NEEDS SUBCATEGORY: None
******************************

--> QUESTION NUMBER _ 985 _ about (MC)


QUESTION: "When reviewing the home care plan for a client with glaucoma, the nurse should encourage him to implement which of the following measures?"

CHOICES

( X ) a.) Reduce daily fluid intake.

( X ) b.) Wear dark glasses in the bright sun.

( X ) c.) Minimize active exercise.

( O ) d.) Add extra lighting to his home.


RATIONALE: Miotic agents may compromise a client's ability to adjust safely to night vision. For safety, extra lighting should be added to the home. The client does not need to curtail fluid intake. Bright lights are not harmful to the eyes. Exercise is permitted, although excessive exertion should be avoided. NURSING PROCESS STEP: Implementation CLIENT NEEDS CATEGORY: Physiological integrity CLIENT NEEDS SUBCATEGORY: Basic care and comfort
******************************

--> QUESTION NUMBER _ 986 _ about (MC)


QUESTION: "The client asks when he can stop taking the eye medication for his chronic open-angle glaucoma. The nurse should tell the client that he:"

CHOICES

( X ) a.) Can stop using the eye drops when his vision improves.

( X ) b.) Needs to use the eye drops only when he has symptoms.

( X ) c.) Can discontinue the eye drops after 2 months of normal eye examinations.

( O ) d.) Must use the eye medication for the rest of his life.


RATIONALE: To control his increased intraocular pressure, the client will need to continue taking eye medications for the rest of his life. Any loss of vision that the client has suffered will be permanent. Vision loss can occur gradually without any symptoms. Intraocular pressure will rise once medications are discontinued. NURSING PROCESS STEP: Implementation CLIENT NEEDS CATEGORY: Physiological integrity CLIENT NEEDS SUBCATEGORY: Pharmacological and parenteral therapies
******************************

--> QUESTION NUMBER _ 987 _ about (MC)


QUESTION: "A client returns from surgery after a submucosal resection with nasal packing in place. Which of the following assessments would be a priority?"

CHOICES

( X ) a.) Determining the degree of pain the client is experiencing.

( O ) b.) Assessing for airway obstruction.

( X ) c.) Observing for ecchymosis in the periorbital region.

( X ) d.) Assessing the client's appetite.


RATIONALE: Postoperative nursing assessment of the client after nasal surgery focuses on early detection of complications. Two common complications are airway obstruction and hemorrhage. The nasal packing can slip out of position and occlude the client's airway. Therefore, assessing the client for airway obstruction is a priority assessment. Assessing pain is important, but assessing for airway obstruction is the highest priority. Ecchymosis will not develop immediately after surgery. Assessing the client's appetite immediately after surgery is not a high priority. NURSING PROCESS STEP: Assessment CLIENT NEEDS CATEGORY: Physiological integrity CLIENT NEEDS SUBCATEGORY: Physiological adaptation
******************************

--> QUESTION NUMBER _ 988 _ about (MC)


QUESTION: "Which of the following techniques is the most appropriate way to assess for posterior nasal bleeding?"

CHOICES

( X ) a.) Change the nasal drip pad frequently and note the amount of drainage.

( X ) b.) Monitor the client's hemoglobin and hematocrit values every 8 hours.

( X ) c.) Frequently assess if the client is nauseated.

( O ) d.) Use a penlight to inspect the back of the pharynx for bleeding.


RATIONALE: The best way for the nurse to detect posterior nasal bleeding is to use a penlight to observe the back of the pharynx. The nasal drip pad will remain dry with posterior nasal bleeding. Checking the client's hemoglobin and hematocrit every 8 hours will not help detect bleeding in its earliest stages. Nausea can occur postoperatively for several reasons; bleeding is just one reason. NURSING PROCESS STEP: Assessment CLIENT NEEDS CATEGORY: Physiological integrity CLIENT NEEDS SUBCATEGORY: Physiological adaptation
******************************

--> QUESTION NUMBER _ 989 _ about (MC)


QUESTION: "After the client returns from surgery for a deviated nasal septum, the nurse would anticipate placing her in what position?"

CHOICES

( X ) a.) Supine.

( X ) b.) Left side-lying.

( O ) c.) Semi-Fowler's.

( X ) d.) Reverse Trendelenburg's.


RATIONALE: To assist in breathing, promote comfort, and decrease edema formation after surgery, the client is most appropriately placed in semi-Fowler's position. The supine position will result in increased swelling, which will increase pain and interfere with breathing. Lying on either side will increase dependent swelling. Reverse Trendelenburg's provides no advantages over semi-Fowler's. NURSING PROCESS STEP: Planning CLIENT NEEDS CATEGORY: Physiological integrity CLIENT NEEDS SUBCATEGORY: Physiological adaptation
******************************

--> QUESTION NUMBER _ 990 _ about (MC)


QUESTION: "Which of the following interventions would likely be most effective for the client to use at home when managing the discomfort of rhinoplasty the initial 2 days after surgery?"

CHOICES

( X ) a.) Applying warm, moist compresses.

( X ) b.) Lying in a prone position.

( X ) c.) Blowing the nose gently.

( O ) d.) Applying ice compresses.


RATIONALE: The most effective way to decrease discomfort is to decrease local edema. Cold application, such as an ice compress or ice bag, is effective. Heat dilates local vessels and increases local congestion. Semi-Fowler's position helps decrease edema and prevent aspiration. Nose blowing should be avoided for at least 48 hours after the nasal packing is removed because it can disrupt the surgical site and lead to bleeding. NURSING PROCESS STEP: Implementation CLIENT NEEDS CATEGORY: Physiological integrity CLIENT NEEDS SUBCATEGORY: Basic care and comfort
******************************

--> QUESTION NUMBER _ 991 _ about (MC)


QUESTION: "The client complains that the nasal packing is uncomfortable and asks when it will be removed. What information should the nurse give the client about the removal of the packing? The nasal packing is usually removed:"

CHOICES

( X ) a.) the day of surgery.

( O ) b.) 24 to 48 hours after surgery.

( X ) c.) after nasal edema subsides.

( X ) d.) after pain has diminished.


RATIONALE: The packing is generally removed the day after surgery. The packing helps maintain hemostasis and prevent bleeding. Removing the packing is uncomfortable and must be done carefully. The client must be watched closely for bleeding after the packing is removed. The packing is generally removed the day after surgery. The purpose of nasal packing is to prevent bleeding. Once hemostasis has been achieved, usually in 24 to 48 hours, the packing can be removed. Nasal edema may not completely resolve for a few weeks. The presence or absence of pain does not influence the timing for removal of nasal packing. NURSING PROCESS STEP: Implementation CLIENT NEEDS CATEGORY: Physiological integrity CLIENT NEEDS SUBCATEGORY: Physiological adaptation
******************************

--> QUESTION NUMBER _ 992 _ about (MC)


QUESTION: "Because the packing blocks the client's nose, a client should be instructed to include which of the following measures after nasal surgery?"

CHOICES

( O ) a.) Frequent mouth care.

( X ) b.) Examine the nares for ulcerations.

( X ) c.) Monitor temperature every 4 hours.

( X ) d.) Normal saline nose drops.


RATIONALE: Mouth-breathing dries the oral mucous membranes. Frequent mouth care is necessary for comfort and to combat the anorexia associated with the taste of blood and loss of the sense of smell. Checking the nares for ulcerations is not necessary. Monitoring the temperature every 4 hours is not necessary. Nose drops are not instilled with packing in place. NURSING PROCESS STEP: Implementation CLIENT NEEDS CATEGORY: Physiological integrity CLIENT NEEDS SUBCATEGORY: Basic care and comfort
******************************

--> QUESTION NUMBER _ 993 _ about (MC)


QUESTION: "Which of the following measures related to food and fluid intake would be appropriate for the client to implement in the early postoperative period after nasal surgery?"

CHOICES

( O ) a.) Increase fluid intake.

( X ) b.) Drink through a straw.

( X ) c.) Take an antiemetic before eating.

( X ) d.) Limit intake of high-fiber foods.


RATIONALE: Although foods as tolerated are encouraged, the nurse should encourage the client with nasal packing to increase fluid intake because fluids are best tolerated at this time. Nasal packing makes eating difficult and uncomfortable. The packing blocks the passage of air through the nose, creating a partial vacuum during swallowing. A sucking action may occur when the client attempts to drink with a straw. Antiemetics are needed only if the client experiences nausea or vomiting. There is no need to limit intake of high-fiber foods. NURSING PROCESS STEP: Implementation CLIENT NEEDS CATEGORY: Physiological integrity CLIENT NEEDS SUBCATEGORY: Basic care and comfort
******************************

--> QUESTION NUMBER _ 994 _ about (MC)


QUESTION: "The nurse would teach the client to implement which of the following nasal care measures after the nasal packing is removed?"

CHOICES

( X ) a.) Irrigate the nares with normal saline solution daily.

( X ) b.) Remove old blood from inside the nares with cotton-tipped applicators.

( O ) c.) Lubricate the membranes with a water-soluble lubricant.

( X ) d.) Avoid cleaning the nares for at least 2 days.


RATIONALE: A water-soluble lubricant offsets dryness and enhances comfort during healing. The lubricant also prevents secretions from drying and crusting in the nose. Irrigating the nares is not recommended as it may initiate bleeding by dislodging clots. The client should be cautioned not to disturb clots because bleeding may occur. Nares may be gently cleaned after removal of the packing. NURSING PROCESS STEP: Implementation CLIENT NEEDS CATEGORY: Physiological integrity CLIENT NEEDS SUBCATEGORY: Basic care and comfort
******************************

--> QUESTION NUMBER _ 995 _ about (MC)


QUESTION: "The nurse should include which of the following information in the client's discharge teaching? "

CHOICES

( O ) a.) The client should expect tarry stools for several days at home.

( X ) b.) Nausea is an expected outcome of surgery and may persist for several days.

( X ) c.) Brief episodes of epistaxis are expected after the surgery.

( X ) d.) The pain from surgery should be resolved within 24 hours of the surgery.


RATIONALE: Nasal bleeding gives stools a tarry appearance for several days; the client should be informed of this effect. Nausea is not expected. Epistaxis is not expected after surgery. Some discomfort can be expected to persist after 24 hours. NURSING PROCESS STEP: Implementation CLIENT NEEDS CATEGORY: Physiological integrity CLIENT NEEDS SUBCATEGORY: Physiological adaptation
******************************

--> QUESTION NUMBER _ 996 _ about (MC)


QUESTION: "A woman tells the nurse at the ambulatory care clinic that her 6-year-old daughter has severe nosebleeds. Which of the following instructions should the nurse give this woman about managing nosebleeds?"

CHOICES

( X ) a.) Help the child assume a comfortable position with her head tilted backward.

( X ) b.) Tilt the child's head backward and place firm pressure on the nose.

( X ) c.) Help the child lie on her stomach and collect the blood on a clean towel.

( O ) d.) Place the child in a sitting position with her neck bent forward and apply firm pressure on the nasal septum.


RATIONALE: For the initial management of nosebleed, the client should sit up and lean forward with the head tipped downward. The soft tissues of the nose should be compressed against the septum with the fingers. The traditional head-back position allows blood to flow down the throat and can trigger vomiting. Tilting the head backward would put the client at risk for aspiration and allow blood to enter the gastrointestinal tract. Tilting the head backward would put the client at risk for aspiration and allow blood to enter the gastrointestinal tract. The client should remain in an upright position. NURSING PROCESS STEP: Implementation CLIENT NEEDS CATEGORY: Safe, effective care environment CLIENT NEEDS SUBCATEGORY: Safety and infection control
******************************

--> QUESTION NUMBER _ 997 _ about (MC)


QUESTION: "Which of the following nursing interventions would be most appropriate for facilitating communication with a client who has a hearing impairment?"

CHOICES

( X ) a.) When speaking, stand to one side of the client and direct the voice directly into the client's ear.

( X ) b.) Stand close to the client and speak as loudly as possible.

( O ) c.) Stand in front of the client and speak slowly and clearly.

( X ) d.) Ask only questions that the client can answer with a "yes" or "no" response.


RATIONALE: Stand close to and directly in front of the client to greatly facilitate communication. Speak slowly and clearly and minimize distractions in the environment. The nurse should face the client; that way the client can see the nurse's mouth at all times for lip-reading. Yelling at the client distorts the voice and further hinders understanding. The nurse should have the client validate his understanding of the conversation by repeating what was said. NURSING PROCESS STEP: Implementation CLIENT NEEDS CATEGORY: Physiological integrity CLIENT NEEDS SUBCATEGORY: Basic care and comfort
******************************

--> QUESTION NUMBER _ 998 _ about (MC)


QUESTION: "Which of the following techniques is appropriate for irrigating an adult client's ear to move cerumen?"

CHOICES

( O ) a.) Allow the irrigating solution to run down the wall of the ear canal.

( X ) b.) Use sterile solution and equipment.

( X ) c.) The irrigating solution should be cool.

( X ) d.) After instilling the solution, pack the ear canal tightly with cotton pledgets.


RATIONALE: The irrigating solution should not be allowed to drop directly on the tympanic membrane because this may cause discomfort or damage. Ear irrigation is considered to be a clean procedure unless the integrity of the tympanic membrane has been damaged. The solution should be at body temperature; when instilled, it should be allowed to run down the side of the ear canal. Cotton pledgets should be placed loosely in the ear canal so they don't exert pressure on the tympanic membrane. NURSING PROCESS STEP: Implementation CLIENT NEEDS CATEGORY: Physiological integrity CLIENT NEEDS SUBCATEGORY: Pharmacological and parenteral therapies
******************************

--> QUESTION NUMBER _ 999 _ about (MC)


QUESTION: "The symptom of vertigo in a client with Meniere's disease is a subjective experience. Which of the following is the most accurate description of vertigo?"

CHOICES

( O ) a.) A feeling that the environment is in motion.

( X ) b.) An episode of blackout.

( X ) c.) Lightheadedness.

( X ) d.) Narrowed vision preceding fainting.


RATIONALE: Vertigo is a form of hallucination in which the person perceives the environment to be moving around him or perceives himself to be moving within the environment. Clients with Meniere's disease do not black out. Clients with Meniere's disease are not light-headed. Clients with Meniere's disease do not faint. NURSING PROCESS STEP: Assessment CLIENT NEEDS CATEGORY: Physiological integrity CLIENT NEEDS SUBCATEGORY: Physiological adaptation
******************************

--> QUESTION NUMBER _ 1000 _ about (MC)


QUESTION: "The client would be experiencing a typical symptom of Meniere's disease if, before an attack, he experienced:"

CHOICES

( X ) a.) A severe headache.

( X ) b.) Blurred vision.

( X ) c.) Nausea.

( O ) d.) A feeling of inner ear fullness.


RATIONALE: Many clients are able to identify an incipient attack of Meniere's disease by a feeling of fullness in the ear that reflects the evolving congestion. Severe headaches are not associated with Meniere's disease. Meniere's disease does not affect vision. Nausea may result after the classic symptom occurs. NURSING PROCESS STEP: Assessment CLIENT NEEDS CATEGORY: Physiological integrity CLIENT NEEDS SUBCATEGORY: Physiological adaptation
******************************